Confusing !!

This topic has expert replies
Legendary Member
Posts: 544
Joined: Thu Oct 08, 2009 9:14 am
Location: Pune, India
Thanked: 31 times
Followed by:2 members

Confusing !!

by adi_800 » Fri Oct 14, 2011 6:18 am
During the past year, Pro-Tect Insurance Company's total payouts on car-theft claims were larger than the company can afford to sustain. Pro-Tect cannot reduce the number of car-theft policies it carries, so cannot protect itself against continued large payouts that way. Therefore, Pro-Tect has decided to offer a discount to holders of car-theft policies whose cars have antitheft devices. Many policyholders will respond to the discount by installing antitheft devices, since the amount of the discount will within two years typically more than cover the cost of installation. Thus, because cars with antitheft devices are rarely stolen, Pro-Tect's plan is likely to reduce its annual payouts.
In the argument above, the two portions in boldface play which of the following roles?
A. The first rules out a certain strategy for achieving a goal; the second presents the strategy that was adopted instead and whose effectiveness the argument assesses.
B. The first is a judgment made in support of a certain conclusion; the second is that conclusion.
C. The first has been used as a consideration to support adopting a certain strategy for achieving a goal; the second reports a decision to adopt an alternative strategy.
D. The first provides evidence in favor of adopting a certain strategy for achieving a goal; the second reports a decision to pursue an alternative goal.
E. The first is a consideration offered against adopting a certain strategy for achieving a goal; the second is the main conclusion that the argument is seeking to establish.

User avatar
Master | Next Rank: 500 Posts
Posts: 496
Joined: Tue Jun 07, 2011 5:34 am
Thanked: 38 times
Followed by:1 members

by sl750 » Fri Oct 14, 2011 9:20 am
IMO A
The goal here is to minimize large payouts. The first strategy of reducing the number of policies is not feasible. The second bold face offers an alternative strategy to reduce the large payouts by offering discounts to holders who have anti-theft devices

Master | Next Rank: 500 Posts
Posts: 217
Joined: Tue May 31, 2011 9:42 pm
Thanked: 8 times
Followed by:2 members

by garima99 » Fri Oct 14, 2011 10:21 am
+1 for A

Legendary Member
Posts: 544
Joined: Thu Oct 08, 2009 9:14 am
Location: Pune, India
Thanked: 31 times
Followed by:2 members

by adi_800 » Fri Oct 14, 2011 10:25 am
Reasons to eliminate B n C??

Master | Next Rank: 500 Posts
Posts: 416
Joined: Thu Jul 28, 2011 12:48 am
Thanked: 28 times
Followed by:6 members

by gunjan1208 » Fri Oct 14, 2011 11:36 pm
I am confused between A and C. But I chose C. what is OA?

Legendary Member
Posts: 544
Joined: Thu Oct 08, 2009 9:14 am
Location: Pune, India
Thanked: 31 times
Followed by:2 members

by adi_800 » Sun Oct 16, 2011 9:53 am
C-> The first has been used as a consideration to support adopting a certain strategy for achieving a goal...

I think the first statement is used to show that why a particular strategy cannot be used and not to support adopting a certain strategy.. This is what the argument says..
Pro-Tect cannot reduce the number of car-theft policies it carries, so cannot protect itself against continued large payouts that way.
So, the first statement gives a reason for NOT adopting a particular strategy and both these combined statements forced Pro-text to decide on the alternative strategy..
So, that's why I guess C is wrong...

OA is A

Master | Next Rank: 500 Posts
Posts: 385
Joined: Fri Sep 23, 2011 9:02 pm
Thanked: 62 times
Followed by:6 members

by user123321 » Wed Nov 23, 2011 8:23 am
+1 for A

user123321
Just started my preparation :D
Want to do it right the first time.

Newbie | Next Rank: 10 Posts
Posts: 6
Joined: Tue Nov 22, 2011 4:59 am

by popy » Wed Nov 23, 2011 8:38 am
why it wouldn't be D?

Master | Next Rank: 500 Posts
Posts: 106
Joined: Sat Mar 02, 2013 4:29 pm
Thanked: 4 times

by buoyant » Tue Nov 05, 2013 10:10 am
What is the official answer?

Need expert help here.

Master | Next Rank: 500 Posts
Posts: 106
Joined: Sat Mar 02, 2013 4:29 pm
Thanked: 4 times

by buoyant » Tue Nov 05, 2013 10:36 am
Looks like A and C are the contenders here.

I think (A) may not be the correct answer.

Answer choice (A) correctly describes the first boldfaced sentence. However,it does not correctly describe the second boldfaced sentence. I think the narration of second boldfaced sentence in choice A is incorrect because it says that the strategy "WAS ADOPTED INSTEAD". passage is talking about "a PLAN to implement such a strategy, but this strategy has not been adopted yet".

I don't understand the first half of the choice (C).
The description of the second boldfaced sentence seems correct in choice C. However,the narration of the first boldfaced sentence doesn't seem right.

Please clarify.

User avatar
GMAT Instructor
Posts: 2193
Joined: Mon Feb 22, 2010 6:30 pm
Location: Vermont and Boston, MA
Thanked: 1186 times
Followed by:512 members
GMAT Score:770

by David@VeritasPrep » Tue Nov 05, 2013 12:14 pm
Very good explanations on this one from sl750, adi800, and edirik and bouyant!

I did some research here. Warning! There are different versions of this question floating around, so be sure to read the answer choices for this particular version.

The source of this question is crack verbal.

OA is A.

It is fairly straightforward to see that the first portion rules out a given strategy. It states that pro-tect cannot adopt the strategy of reducing the number of car-theft policies.

This rules out choice B, C, and D. E is still possible since E states that it is "a consideration against adopting a certain strategy. However, choice E is certainly wrong on the second portion. This is not the main conclusion but is a different strategy that the argument then discusses.
Veritas Prep | GMAT Instructor

Veritas Prep Reviews
Save $100 off any live Veritas Prep GMAT Course

GMAT/MBA Expert

User avatar
GMAT Instructor
Posts: 768
Joined: Wed Dec 28, 2011 4:18 pm
Location: Berkeley, CA
Thanked: 387 times
Followed by:140 members

by Mike@Magoosh » Tue Nov 05, 2013 1:44 pm
buoyant wrote:Hi Mike,
Can you please respond to the below thread to answer and explain GMATPrep question? There is so much confusion around this question and the official answer. Thanks!
Dear bouyant,
I'm happy to help. :-)

Here is an article about Boldface CR questions, with 3 practice questions:
https://magoosh.com/gmat/2013/gmat-criti ... questions/

Here's my analysis of this question.
A. The first rules out a certain strategy for achieving a goal; the second presents the strategy that was adopted instead and whose effectiveness the argument assesses.
This one looks promising. The first definitely is a strategy that won't work. The second is a potential solution, and the argument does explore its potential. This could be a legitimate answer to the question.

B. The first is a judgment made in support of a certain conclusion; the second is that conclusion.
No. The first is the rejection of a road they didn't take and didn't discuss further.

C. The first has been used as a consideration to support adopting a certain strategy for achieving a goal; the second reports a decision to adopt an alternative strategy.
Hmmm. The first does consider a certain strategy, but doesn't "support" it --- it decides against it, saying that it would be impossible to pursue it.

D. The first provides evidence in favor of adopting a certain strategy for achieving a goal; the second reports a decision to pursue an alternative goal.
Similar to (C), the first is a reason why the company can't do something. It's certainly not evidence in favor of doing it.

E. The first is a consideration offered against adopting a certain strategy for achieving a goal; the second is the main conclusion that the argument is seeking to establish.
This one works with the first bold statement, but here the second bold statement doesn't fit. The second bold statement is a decision to do something, but what follows, what they hope to achieve from implementing that decision --- that's really the conclusion of the argument. "Thus .... Pro-Tect's plan is likely to reduce its annual payouts." That's the conclusion, not the second bold statement.

The only plausible answer (A).

Does all this make sense?
Mike :-)
Magoosh GMAT Instructor
https://gmat.magoosh.com/

Master | Next Rank: 500 Posts
Posts: 106
Joined: Sat Mar 02, 2013 4:29 pm
Thanked: 4 times

by buoyant » Tue Nov 05, 2013 2:57 pm
Thanks David and Mike for the explanation!

I am still not clear why does choice A say that "the second presents a strategy that WAS ADOPTED instead"..
Doesn't the passage say that "Pro-Tect has a Plan to do so and later discuss the possible advantages of this strategy after it will be adopted" ?

GMAT/MBA Expert

User avatar
GMAT Instructor
Posts: 768
Joined: Wed Dec 28, 2011 4:18 pm
Location: Berkeley, CA
Thanked: 387 times
Followed by:140 members

by Mike@Magoosh » Tue Nov 05, 2013 4:16 pm
buoyant wrote:Thanks David and Mike for the explanation!

I am still not clear why does choice A say that "the second presents a strategy that WAS ADOPTED instead"..
Doesn't the passage say that "Pro-Tect has a Plan to do so and later discuss the possible advantages of this strategy after it will be adopted" ?
Dear buoyant,
When I "adopt a strategy", that means I have formally made the decision to follow that strategy. It doesn't necessarily mean that I have done anything beyond the decision ---- I might not have implemented even step #1 yet. All I have done is made a decision. The second bold statement explicitly says that a decision has been made. Therefore, this strategy has been adopted.

Does this make sense?
Mike :-)
Magoosh GMAT Instructor
https://gmat.magoosh.com/

Master | Next Rank: 500 Posts
Posts: 106
Joined: Sat Mar 02, 2013 4:29 pm
Thanked: 4 times

by buoyant » Wed Nov 06, 2013 8:01 am
Well explained, Mike !

It is clear now.

Thanks!